0

Division of a fraction - class-VIII

Attempted 0/46 Correct 0 Score 0

Evaluate: $\dfrac{(3\dfrac{2}{3})^{2} -(2\dfrac{1}{2})^{2}}{(4\dfrac{3}{2})^{2} -(3\dfrac{1}{3})^{2}}$ $\div$ $\dfrac{3\dfrac{2}{3} -2\dfrac{1}{2}}{4\dfrac{3}{2} -3\dfrac{1}{3}}$

  1. $\dfrac{35}{53}$

  2. $\dfrac{37}{53}$

  3. $\dfrac{42}{59}$

  4. $\dfrac{47}{60}$


Correct Option: B
Explanation:

$\dfrac{(3\frac{2}{3})^2-(2\frac{1}{2})^2}{(4\frac{3}{2})^2-(3\frac{1}{3})^2} \div \dfrac{3\frac{2}{3}-2\frac{1}{2}}{4\frac{3}{2}-3\frac{1}{3}}$


$=\dfrac{(3\frac{2}{3}+2\frac{1}{2})(3\frac{2}{3}-2\frac{1}{2})}{(4\frac{3}{2}+3\frac{1}{3})(4\frac{3}{2}-3\frac{1}{3})} \div \dfrac{3\frac{2}{3}-2\frac{1}{2}}{4\frac{3}{2}-3\frac{1}{3}}$

$=\dfrac{3\frac{2}{3}+2\frac{1}{2}}{4\frac{3}{2}+3\frac{1}{3}}$

$=\dfrac{\dfrac{11}{3}+\dfrac{5}{2}}{\dfrac{11}{2}+\dfrac{10}{3}}$

$=\dfrac{\dfrac{22+15}{6}}{\dfrac{33+20}{6}}$

$=\dfrac{37}{53}$

Solve $\left[\dfrac{170}{3} +\dfrac{6}{7}\right] \div \left[\dfrac{2}{7} \times \dfrac{11}{2}\right]$

  1. $\dfrac{1208}{3\times 11}$

  2. $\dfrac{1208}{11}$

  3. $\dfrac{1208}{3}$

  4. $\dfrac{1208}{9\times 11}$


Correct Option: A
Explanation:

$\left[\dfrac{170}{3} +\dfrac{6}{7}\right] \div \left[\dfrac{2}{7} \times \dfrac{11}{2}\right]$


$=\left[ \dfrac{1190+18}{21}\right] \div \left[ \dfrac{11}{7}\right]$


$=\left[ \dfrac{1190+18}{21}\right] \times \left[ \dfrac{7}{11}\right]$

$=\left[ \dfrac{1208}{11\times 3}\right]$

Divide the difference of $\dfrac{1}{5}$ and $\dfrac{2}{7}$ by $\dfrac{2}{7}$.

  1. $\dfrac{1}{10}$

  2. $\dfrac{21}{10}$

  3. $\dfrac{3}{10}$

  4. $\dfrac{4}{10}$


Correct Option: C
Explanation:

Difference of  $\dfrac{1}{5}$  and $\dfrac{2}{7}$ is  


$\dfrac{2}{7} - \dfrac{1}{5} = \dfrac{2\times5 - 1\times7}{35} =\dfrac{10 - 7}{35} =\dfrac{3}{35}$

Now divide $\dfrac{3}{35}$  by $\dfrac{2}{7}$  

we have, $\dfrac{3}{35}÷\dfrac{2}{7} =\dfrac{3}{35}\times \dfrac{7}{2} =\dfrac{3}{10}$

Ans: $\dfrac{3}{10}$

Simplify $35\times 6\dfrac{1}{14}$(approximately)$=$

  1. $220.5$

  2. $220$

  3. $212$

  4. $231$


Correct Option: C
Explanation:
$6 \cfrac{1}{14} = \cfrac{14 \times 6 + 1}{14} = \cfrac{85}{14}$
$\therefore \; 35 \times 6\cfrac{1}{14} = 35 \times \cfrac{85}{14} = \cfrac{5 \times 85}{2} = \cfrac{425}{2} = 212.5 \approx 212$
Hence, 212 is the correct answer.

A ribbon of length $5\dfrac{1}{4}$m is cut in to small pieces each of length $\dfrac{3}{4}$ m number of pieces will be

  1. $5$

  2. $6$

  3. $7$

  4. $8$


Correct Option: C
Explanation:

$ \Rightarrow  $ let length l $ = 5\dfrac{1}{4}$ m 

$ l = 5+\dfrac{1}{4} = \dfrac{21}{4}m $

& Small n pieces of length $ x = \dfrac{3}{4}$m 

So, $ l = nx $

$ n = \dfrac{l}{x} = \dfrac{21}{4}\times \dfrac{4}{3} = 7 $

So, there are 7 pieces of length $ \dfrac{3}{4} $

The unit digit of $2017^{2017}$.

  1. $7$

  2. $9$

  3. $3$

  4. $1$


Correct Option: A
Explanation:

Consider the unit digits of $7^{x}$ series which is $7,9,3,1$

These four digits go on repeat So consider Remainder of ${2017}\div{4}=1$
so unit digit is $7$

If $Rs.510$ be divided among $A,B,C$ in such a way that $A$ gets $\dfrac{2}{3}$ of what $B$ gets and $B$ gets $\dfrac{1}{4}$ of what $C$ gets, then their shares are respectively:

  1. $Rs.120, Rs.240, Rs.150$

  2. $Rs.60, Rs.90, Rs.360$

  3. $Rs.150, Rs.300, Rs.60$

  4. $None\ of\ these$


Correct Option: B
Explanation:
$A=\dfrac{2}{3}B$

$B=\dfrac{1}{4}C$

$\Rightarrow C=4B$

$A+B+C=510$                                            

$=2B/3+B+4B=510$
                               
$=2B+3B+12B=510\times 3$                      

$=17B=510\times 3$ 
                                             
$B=30\times 3=90$

$A=\dfrac{2\times 90}{3}$

$=60$

 $C=4\times 90$

$=360$.

Evaluate: 

$(5 \div 2.25) \div (9 \div 2.25)$.

  1. $\dfrac{5}{9}$

  2. $\dfrac{100}{9}$

  3. $\dfrac{9}{5}$

  4. $1$


Correct Option: A
Explanation:

$ \dfrac {\dfrac {5}{2.25}}{\dfrac {9}{2.25}} = \dfrac {5}{9}$

Simplify : $\displaystyle \frac{2+2\times 2}{2\div 2\times 2}\div \frac{\frac{1}{2}\div \frac{1}{2} \, \text{of} \, \frac{1}{2}}{\frac{1}{2}+\frac{1}{2} \, \text{of} \, \frac{1}{2}}$

  1. $1$

  2. $2$

  3. $\displaystyle 1\frac{1}{3}$

  4. $\displaystyle 1\frac{1}{8}$


Correct Option: D
Explanation:

Given exp =$\displaystyle =\frac{2+4}{1\times 4}\div \frac{\frac{1}{2}\div \frac{1}{4}}{\frac{1}{2}+\frac{1}{4}}$
$\displaystyle =\frac{6}{2}\div\frac{\frac{1}{2}\times \frac{4}{1}}{\frac{3}{4}}=3\div \frac{2}{\frac{3}{4}} $
$\displaystyle =3\div \frac{8}{3}=3\times \frac{3}{8}=\frac{9}{8}=1\frac{1}{8}$

Find the value of $ \cfrac{2}{3}\times \cfrac{3}{\tfrac{5}{6}\div \tfrac{2}{3} \, \text {of} \, \,1\tfrac{1}{4}}$

  1. $2$

  2. $1$

  3. $\displaystyle \frac{1}{2}$

  4. $\displaystyle \frac{2}{3}$


Correct Option: A
Explanation:

Given expression

$ \cfrac{2}{3}\times \cfrac{3}{\tfrac{5}{6}\div \tfrac{2}{3} \, \text {of} \, \,1\tfrac{1}{4}}$

$=\displaystyle \frac{2}{3}\times\dfrac{3}{\tfrac{5}{6}\div \left ( \tfrac{2}{3}\times\tfrac{5}{4} \right )}$


$\displaystyle=\frac{2}{3}\times\dfrac{3}{\tfrac{5}{6}\div \tfrac{5}{6}}=\frac{2}{3}\times\frac{3}{1}=2$

What is the simplified value of $\displaystyle \frac{\frac{1}{3}\div \frac{1}{3}\times\frac{1}{3}}{\frac{1}{3}\div \frac{1}{3}of\frac{1}{3}}-\frac{1}{9}?$

  1. $0$

  2. $2$

  3. $\dfrac{1}{9}$

  4. $\dfrac{2}{9}$


Correct Option: A
Explanation:

(Using BODMAS)
$\displaystyle \frac{\frac{1}{3}\div \frac{1}{3}\times \frac{1}{3}}{\frac{1}{3}\div \frac{1}{3}of\frac{1}{3}}-\frac{1}{9}=\frac{\frac{1}{3}\times \frac{3}{1}\times \frac{1}{3}}{\frac{1}{3}\div \frac{1}{9}}-\frac{1}{9}$
$\displaystyle =\frac{1\times \frac{1}{3}}{\frac{1}{3}\times \frac{9}{1}}-\frac{1}{9}=\frac{1}{3\times 3}-\frac{1}{9}=\frac{1}{9}-\frac{1}{9}=0$

Sunny was given $\displaystyle \frac{1}{3}$ of a sum of money and Ankur was given $\displaystyle \frac{1}{3}$ of what was left. What is Ankur's share as a fraction of Sunny's Share?

  1. $\displaystyle \frac{2}{9}$

  2. $\displaystyle \frac{1}{3}$

  3. $\displaystyle \frac{2}{3}$

  4. $\displaystyle \frac{1}{9}$


Correct Option: C
Explanation:
Sunny gives $\dfrac { 1 }{ 3 }$ of his money
Left amount will be $ 1-\dfrac { 1 }{ 3 } = \dfrac { 2 }{ 3 }$ 
Amount received by Ankur is $\dfrac { 1 }{ 3 }$ of left amount $(\frac { 2 }{ 3 }$  of total amount $)$ 
Amount received by Ankur will be $\dfrac { 1 }{ 3 } \times \dfrac { 2 }{ 3 }$ of total amount $= \dfrac { 2 }{ 9 }$ of total amount
Ankur's share as a fraction of Sunny's share is $\dfrac{\frac{2}{9}}{\frac{1}{3}}=\dfrac{2}{3}$
So Correct answer will be option C

If we multiply a fraction by itself and divide the product by its reciprocal the fraction thus 
obtained is $\displaystyle 18\frac{26}{27}$. The original fraction is 

  1. $\displaystyle \frac{8}{27}$

  2. $\displaystyle 2\frac{2}{3}$

  3. $\displaystyle 1\frac{1}{2}$

  4. None of these


Correct Option: B
Explanation:

$Let\quad fraction\quad x\quad then\ x\times x\div \dfrac { 1 }{ x } =18\dfrac { 26 }{ 27 } =\dfrac { 512 }{ 27 }$


 $x\times x\times \dfrac { x }{ 1 } =\dfrac { 512 }{ 27 } \ we\quad get\quad x=\dfrac { 8 }{ 3 } =2\dfrac { 2 }{ 3 } $
So correct answer will be option B

Find the value of $\displaystyle \frac{2}{1+\frac{1}{1-\frac{1}{2}}}\times\frac{3}{\frac{5}{6}of\frac{3}{2}\div 1\frac{1}{4}}$

  1. $2$

  2. $5$

  3. $6$

  4. $1$


Correct Option: A
Explanation:

$\displaystyle \frac{2}{\displaystyle1+\frac{1}{\displaystyle1-\frac{1}{\displaystyle2}}}\times\frac{3}{\displaystyle\frac{5}{\displaystyle6}of\frac{3}{\displaystyle2}\div 1\frac{1}{\displaystyle4}}$
$=\displaystyle \frac{2}{\displaystyle1+\frac{1}{\displaystyle\frac{1}{\displaystyle2}}}\times\frac{3}{\displaystyle\frac{15}{\displaystyle12}\div \frac{5}{\displaystyle4}}$

$=\displaystyle\frac{2}{\displaystyle1+2}\times\frac{3}{\displaystyle\frac{15}{\displaystyle12}\times\frac{4}{\displaystyle5}}=\frac{2}{\displaystyle3}\times\frac{3}{\displaystyle1}=2$

When any number is divided by 1, the quotient is

  1. 1

  2. 0

  3. 2

  4. number itself


Correct Option: D
Explanation:

Any number  divided by 1 equals that number.
for eg:
$8 \div 1 = 8$
$10 \div 1 = 10$
$\therefore $When any number is divided by 1, the quotient is number itself.
Option D is correct.

Simplify : $\displaystyle \left [ 3\frac{1}{4}\div \left { 1\frac{1}{4}-\frac{1}{2}\left ( 2\frac{1}{2}-\frac{1}{4}-\frac{1}{6} \right ) \right } \right ]\div \left ( \frac{1}{2}of4\frac{1}{3} \right )$

  1. 18

  2. 36

  3. 39

  4. 78


Correct Option: B
Explanation:

Given exp.
$\displaystyle =\left [ \frac{13}{4}\div \left { \frac{5}{4}-\frac{1}{2}\left ( \frac{5}{2}-\frac{3\, \, \,2}{2} \right ) \right } \right ]\div \left ( \frac{1}{2}of\frac{13}{3} \right )$
$\displaystyle =\left [ \frac{13}{4}\div \left { \frac{5}{4}-\frac{1}{2}\left ( \frac{5}{2}-\frac{1}{12} \right ) \right } \right ]\div \frac{13}{6} $
$=\displaystyle \left [ \frac{13}{4}\div \left { \frac{5}{4}-\frac{1}{2}\times \frac{30-1}{12} \right } \right ]\div \frac{13}{6}$
$\displaystyle =\left [ \frac{13}{4}\div \left { \frac{5}{4}-\frac{29}{24} \right } \right ]\div \frac{13}{6}$
$\displaystyle =\left [ \frac{13}{4}\div \frac{30-29}{24} \right ]\div \frac{13}{6}$
$\displaystyle =\left ( \frac{13}{4} \div \frac{1}{24}\right )\div \frac{13}{4}=\frac{13}{4}\times 24\times \frac{6}{13}=36$

If $2805\,\div\, 2.55\, =\, 1100\,, then\, 280.5\,\div\,25.5\, =\,$ .............

  1. 1.1

  2. 1.01

  3. 0.11

  4. 11


Correct Option: D
Explanation:

$\displaystyle\frac{280.5}{25.5}\,=\, \frac{280.5}{25.5}\,\times\, \frac{10}{10}\,\times\, \frac{10}{10}$

$=\,\displaystyle\frac{2805}{2.55}\,\times\,\frac{1}{100}$

$=\,\displaystyle\frac{1100}{100}\,=\, 11$

$143.6\, \div\, 2000\, =\,..........$

  1. 0.1718

  2. 7.18

  3. 0.718

  4. 0.0718


Correct Option: D
Explanation:

$143.6\div 200=71.8\div100=0.0718$

 Hence the answer is option is D

15 of $\displaystyle \frac{1}{5}$ is

  1. $\displaystyle \frac{1}{75}$

  2. $\displaystyle \frac{151}{5}$

  3. 3

  4. -3


Correct Option: C
Explanation:

15 of $\displaystyle \frac{1}{5}\, =\, 15\, \times\, \displaystyle \frac{1}{5}\, =\, 3$

If $\sqrt{5}=2.236$ and $\sqrt{10}=3.162$, the value of $\displaystyle\frac{\sqrt{10}-\sqrt{5}}{\sqrt{2}}$ on simplifying is

  1. 0.455

  2. 0.855

  3. 0.655

  4. 0.755


Correct Option: C
Explanation:

values of $\sqrt { 5 } $ and $\sqrt { 10 } $ are given.

as per problem,
$\frac { \sqrt { 10 } -\sqrt { 5 }  }{ \sqrt { 2 }  } $
$\frac { 3.162-2.236 }{ 1.414 } $(value of $\sqrt { 2 } $ is1.414)
$\frac { 0.926 }{ 1.414 } $
$0.655$

The value of $\displaystyle\frac{1}{\sqrt{3}+\sqrt{2}-1}$ on simplifying upto 3 decimal places, given that $\sqrt{2}=1.4142$ and $\sqrt{6}=2.4495$ is

  1. 0.166

  2. 0.366

  3. 0.466

  4. 0.566


Correct Option: C
Explanation:

$\frac { 1 }{ \sqrt { 3 } +\sqrt { 2 } -1 } $
$=\frac { \sqrt { 3 } -\left( \sqrt { 2 } -1 \right)  }{ \left( \sqrt { 3 } +\left( \sqrt { 2 } -1 \right)  \right) \left( \sqrt { 3 } -\left( \sqrt { 2 } -1 \right)  \right)  } Multiplying\sqrt { 3 } -\left( \sqrt { 2 } -1 \right) \quad with\quad numerator\quad and\quad denominator\quad $
$=\frac { \sqrt { 3 } -\left( \sqrt { 2 } -1 \right)  }{ 3-{ \left( \sqrt { 2 } -1 \right)  }^{ 2 } } $
 $=\frac { \sqrt { 3 } -\left( \sqrt { 2 } -1 \right)  }{ 3-{ \left( 2+1-2\sqrt { 2 }  \right)  } } $
 $=\frac { \sqrt { 3\quad  } -\sqrt { 2 } +1 }{ 2\sqrt { 2 }  } $
$=\frac { 1.732-1.4142+1 }{ 2.8284 } =0.466$
                                               $(\sqrt { 3 } =\frac { \sqrt { 6 }  }{ \sqrt { 2 }  } =\frac { 2.4495 }{ 1.4142 } =1.732)$

Place value chart is extended on .............. side to provide place for fractions

  1. right

  2. left

  3. no

  4. None of these


Correct Option: A
Explanation:

That is the role of the decimal point. The decimal point separates the place values that are whole values on the left from the place values that are fractional parts on the right.

So option A is the correct answer.

If $\displaystyle 2805\div 2.55=1100$ then $\displaystyle 280.5\div 25.5=$ _______

  1. 1.1

  2. 1.01

  3. 0.11

  4. 11


Correct Option: D
Explanation:

$\displaystyle \frac{280.5}{25.5}=\frac{280.5}{25.5}\times \frac{10}{10}\times \frac{10}{10}$

$\displaystyle =\frac{2805}{2.55}\times \frac{1}{100}$

$\displaystyle =\frac{1100}{100}=11$

The fraction $\displaystyle \frac{9}{4}$ can be written as

  1. $\displaystyle \dfrac{\dfrac{9}{2}}{\dfrac{4}{2}}$

  2. $\displaystyle \dfrac{\dfrac{9}{4}}{1}$

  3. $\displaystyle \dfrac{\dfrac{9}{5}}{\dfrac{4}{5}}$

  4. $\displaystyle \dfrac{\dfrac{7}{6}}{\dfrac{9}{4}}$


Correct Option: A,B,C
Explanation:
$ \dfrac{\dfrac{9}{2}}{\dfrac{4}{2}} = \dfrac{9}{2} \div \dfrac 42 =\dfrac 92 \times \dfrac 24  = \dfrac 94$

$ \dfrac{\dfrac{9}{4}}{1} = \dfrac{9}{4} \div 1 = \dfrac 92 \times 1 = \dfrac 94$

$ \dfrac{\dfrac{9}{5}}{\dfrac{4}{5}} = \dfrac{9}{5} \div \dfrac 45 =\dfrac 95 \times \dfrac 45  = \dfrac 94$

So, options $A, B$ and $C$ are correct.

Which of the following is complex fraction?

  1. $\dfrac{6\dfrac{1}{3}}{9}$

  2. $\dfrac{4}{9}$

  3. $\dfrac{5}{9}$

  4. $\dfrac{8}{9}$


Correct Option: A
Explanation:

$\dfrac{6\dfrac{1}{3}}{9}$ is complex fraction.


So, option A is correct.

Divide $\dfrac35$ by $4$.

  1. $\dfrac{12}5$

  2. $\dfrac3{20}$

  3. $\dfrac{12}{20}$

  4. $\dfrac35$


Correct Option: B
Explanation:
According to problem

$ \dfrac{3}{5}  \div 4$

$  = \dfrac{3}{5} \times \dfrac14$

$=  \dfrac3{20} $

So option $B $ is correct

Divide $\dfrac{25}{36}$ by $5$.

  1. $\dfrac{5}{6}$

  2. $\dfrac{25}{6}$

  3. $\dfrac{5}{36}$

  4. $\dfrac{36}{5}$


Correct Option: C
Explanation:

Given that 

divide $\dfrac{25}{36} $ by $5$, writing the expression

$\dfrac{25}{36}  \div 5$

$ = \dfrac{25}{36}  \times  \dfrac15$

$ = \dfrac{5}{36}  $

So option $C $ is correct

Divide $\dfrac45$ by $2$.

  1. $\dfrac25$

  2. $\dfrac15$

  3. $\dfrac85$

  4. None of these


Correct Option: A
Explanation:

$\dfrac{4}{5} \div 2 = \dfrac{\dfrac{4}{5}}{2} = \dfrac{4}{10}$


On simplifying, dividing numerator and denominator by $2$, we get $\dfrac{2}{5}$

If we divide $1$ by a fraction $x$, we get ______ $x$.

  1. same fraction as

  2. reciprocal of

  3. double of

  4. half of


Correct Option: B
Explanation:

Every number has a reciprocal except 0 $(\dfrac{1}{0}$ is undefined$)$. The reciprocal is shown as $\dfrac{1}{x}$. 


When we multiply a number by its reciprocal we get $1$

$\Rightarrow$ If we divide $1$ by $x$, we get reciprocal of $x$.

Divide $10$ by $\dfrac{20}{19}$.

  1. $\dfrac{19}{2}$

  2. $\dfrac{1}{20}$

  3. $\dfrac{19}{20}$

  4. $\dfrac{9}{2}$


Correct Option: A
Explanation:
According to problem
$10$   $\div \dfrac{20}{19}$

$  = 10 \times \dfrac{19}{20}$
$=  \dfrac{19}2 $

Simplify the following:
$\dfrac {3}{8} \div \dfrac {24}{48} =\dfrac{3}{4}$

  1. True

  2. False


Correct Option: A
Explanation:
$\dfrac{3}{8} \div \dfrac{{24}}{{48}} = \dfrac{3}{8} \times \dfrac{{48}}{{24}} = \dfrac{3}{4}$
Hence, the simplification is true.

Divide the sum of $\displaystyle\frac{65}{12}$ and $\displaystyle\frac{12}{7}$ by their difference.

  1. $\displaystyle\frac{599}{311}$

  2. $\displaystyle\frac{680}{216}$

  3. $\displaystyle\frac{642}{133}$

  4. $\displaystyle\frac{501}{301}$


Correct Option: A
Explanation:

sum of $\dfrac{65}{12}$  and $\dfrac{12}{7} $


$ \dfrac{65}{12} +\dfrac {12}{7}= \dfrac{65\times 7 +12\times 12}{12\times 7} $

                  $=\dfrac{599}{84}$ ......................(1)

difference of  $\dfrac{65}{12}$  and $\dfrac{12}{7} $

$\dfrac{65}{12}-\dfrac{12}{7} = \dfrac{65\times 7 - 12\times 12}{84}= \dfrac{311}{84} $

                  $ =\dfrac{311}{84} $.........................(2)

divide the sum with difference i.e.

dividing equation (1) with equation (2)

$=\dfrac{\dfrac{599}{84}}{\dfrac{311}{84}}$

$=\dfrac{599}{311}$

Divide $\dfrac{15}{38}$ by $\dfrac{-3}{19}$

  1. $\dfrac{-2}{5}$

  2. $\dfrac{-5}{2}$

  3. $\dfrac{2}{5}$

  4. $\dfrac{5}{2}$


Correct Option: B
Explanation:
$\dfrac{15}{38} \div \dfrac{-3}{19}$

$=\dfrac{15\times19}{38 \times -3}$

$=\dfrac{-5}{2}$

$\left(\large{\frac{-5}{3}}\right)^5$ $\div$ $\left(\large{\frac{-5}{3}}\right)^{7}$

  1. $\large{\frac{25}{9}}$

  2. $\large{\frac{9}{25}}$

  3. $\large{\frac{16}{25}}$

  4. $\large{\frac{25}{16}}$


Correct Option: B
Explanation:

$\left(\large{\frac{-5}{3}}\right)^5$ $\div$ $\left(\large{\frac{-5}{3}}\right)^{7}$


$=-\left(\large{\frac{5}{3}}\right)^5$ $\times$ $-\left(\large{\frac{3}{5}}\right)^{7}$


$=\left(\dfrac{3}{5}\right)^2$

$=\dfrac{9}{25}$.

Evaluate: $\dfrac {\left(\dfrac {-3}{5}\right)^{3} \times \left(\dfrac {9}{25}\right)^{2} \times \left(\dfrac {-18}{125}\right)^{o}}{\left(\dfrac {-27}{125}\right) \times \left(\dfrac {-3}{5}\right)}$

  1. $\dfrac{27}{125}$

  2. $-\dfrac{27}{125}$

  3. $\dfrac{64}{125}$

  4. $-\dfrac{64}{125}$


Correct Option: B
Explanation:

$\dfrac {\left(\dfrac {-3}{5}\right)^{3} \times \left(\dfrac {9}{25}\right)^{2} \times \left(\dfrac {-18}{125}\right)^{0}}{\left(\dfrac {-27}{125}\right) \times \left(\dfrac {-3}{5}\right)}=\dfrac{\dfrac{-3^3\times 3^4\times 1}{5^3\times 5^4\times 1}}{\dfrac{3^3\times 3}{5^3\times 5}}=-\dfrac{3^{7-4}}{5^{7-4}}=-\dfrac{27}{125}$

Convert the following into fraction.
$22.5\%$

    • $\dfrac{9}{40}$
  1. $\dfrac{225}{100}$

  2. $\dfrac{22}{100}$

  3. $\dfrac{5}{100}$


Correct Option: A
Explanation:
     $22.5$%
$=\dfrac{22.5}{100}$
$=\dfrac{48}{2}\times \dfrac{1}{100}$
$=\dfrac{9}{40}$

Evaluate the following :

$\displaystyle  8 \times \dfrac {\dfrac{5}{24}}{\dfrac {7}{12}} $.

  1. $\displaystyle \frac {5 }{24}$

  2. $\displaystyle \frac {20}{7}$

  3. $\displaystyle \frac {2}{7}$

  4. $\displaystyle \frac {21}{5}$


Correct Option: B
Explanation:

Given, $ 8 \times \displaystyle \frac {\frac {5}{24}}{\frac {7}{12}} $

$ \therefore 8 \times \displaystyle \frac {\frac {5 }{24}}{\frac {7}{12}} = 8 \times \frac {5\times 12}{7 \times 24}   $

$=\displaystyle \frac {5 \times 4}{7}$

$= \displaystyle \frac {20}{7} $

$I = \displaystyle \frac{3}{4}\div \frac{5}{6}$ 

$II = 3\displaystyle \div $[($4\displaystyle \div 5$)$\displaystyle \div 6$]
$III = [3\displaystyle \div (4\displaystyle \div 5)]\displaystyle \div 6$
$IV = 3\displaystyle \div 4(5\displaystyle \div 6)$
Select the correct option from the following.

  1. I and II are equal

  2. I and IV are equal

  3. I and III are equal

  4. All are equal


Correct Option: B
Explanation:

$ I=\cfrac{3}{4}\times \cfrac{6}{5}=\cfrac{9}{10}$


$II=3\div \left [ \cfrac{4}{5}\times \cfrac{1}{6} \right ]=3\times \cfrac{15}{2}=\cfrac{45}{2}$

$III=\left [ 3\div \cfrac{4}{5} \right ]\div 6=3\times \cfrac{5}{4}\times \cfrac{1}{6}=\cfrac{5}{8}$

$ IV=3\div 4\times\cfrac{5}{6}=3\times \cfrac{3}{10}=\cfrac{9}{10}$

Hence, $I$ and $IV$ are equal.

The least fraction that must be added to $\displaystyle1\frac{1}{3}\div 1\frac{1}{2}\div 1\frac{1}{9}$  to make the result an integer is:

  1. $\displaystyle\frac{4}{5}$

  2. $\displaystyle\frac{3}{5}$

  3. $\displaystyle\frac{2}{5}$

  4. $\displaystyle\frac{1}{5}$


Correct Option: D
Explanation:

$\displaystyle 1\frac{1}{3}\div 1\frac{1}{2}=\frac{4}{3}\div \frac{3}{2}\div \frac{10}{9}$
$\displaystyle =\frac{4}{3}\times \frac{2}{3}\div \frac{10}{9}=\frac{8}{9}\times \frac{9}{10}=\frac{4}{5}$
$\displaystyle \therefore \frac{1}{5}$ should be added to $\displaystyle \frac{4}{5}$ to make it an integer

The number of positive fractions m/n such that $1/3< m/n < 1$ and having the property that the fraction remains the same by adding some positive integer to the numerator and multiplying the denominator by the same positive integer is

  1. $1$

  2. $3$

  3. $6$

  4. infinite


Correct Option: B
Explanation:

$m, n$ are integer
$1/3 < m/n < 1 $...(i)
$\displaystyle \frac{m + \alpha}{n \alpha} = \frac{m}{n}        n \neq 0$
$\alpha = \displaystyle \frac{m}{m - 1} = +$ve integer
$\therefore m = 2$
Using equation (i)
$n = 3, 4, 5$
$\therefore \displaystyle \frac{m}{n}= \frac{2}{3} , \frac{2}{4}, \frac{2}{5}$

$\dfrac{25 \% \, of  \ 50\% \ of \ 100 \%}{25 \,of \,100 \times 50 \%\, of \,100 }$ is equal to ______ .

  1. $ 0.0001\%$

  2. $ 0.1\%$

  3. $ 0.01\%$

  4. $ 1\%$


Correct Option: D
Explanation:

Given $\cfrac {25\%\quad of\quad 50\%\quad of\quad 100\%}{25\quad of\quad 100\times 50\%\quad of\quad 100}$

So, first solving solving numerator
$\cfrac {25}{100}$ of $\cfrac {50}{100}$ of $\cfrac {100}{100}$ 
$\implies \cfrac {25}{100}\left(\cfrac {50}{100}\left(\cfrac {100}{100}\right)\right)\quad equation (1)$  
Now taking denominator
$25$ of $100\times 50\%$ of $100$ 
So, $25$ of $100$ means $\cfrac {25}{100}$ which is $\cfrac {1}{4}$
$50\%$ of $100=\cfrac {50}{100}\times 100=50$
So, denominator becomes $\cfrac {1}{4}\times 50\quad equation (2)$
$\cfrac {Numerator}{Denominator}=\cfrac {equation (1)}{equation (2)}$
$=\cfrac {\cfrac {25}{100}\left(\cfrac{50}{100}\left(\cfrac{100}{100}\right)\right)}{\cfrac {1}{4}\times 50}$
$=\cfrac {\cfrac {1}{4}\times \cfrac {1}{2}}{\cfrac {1}{4}\times 50}$
$=\cfrac {1}{2}\times 50$
In terms of percent $=\cfrac {1}{100}\times 100=1\%$

The least number among $\displaystyle \frac{4}{9}, \, \sqrt{\frac{9}{49}},$ 0.45 and $(0.8)^2$ is

  1. $\displaystyle \frac{4}{9}$

  2. $\displaystyle \sqrt{\frac{9}{49}}$

  3. 0.45

  4. $(0.8)^2$


Correct Option: B
Explanation:
: Decimal equivalents of the given numbers.
$\displaystyle \frac{4}{9} \, = \, 0.44; \, \sqrt{\frac{9}{49}} \, = \, \frac{3}{7} \, = \, 0.43$
$\displaystyle 0.45 \, and \, (0.8)^2 \, = \, 0.64$
$\displaystyle \therefore$ Least number is 0.43
$\displaystyle = \, \sqrt{\frac{9}{49}}$

If $a=3567, b=10, c=100, d=1000$ & $e=10000$ then $\frac{a}{b}+\frac{a}{c}-\frac{a}{d}+\frac{a}{e}$ is less than

  1. $3.962937$

  2. $3962.937$

  3. $39.62937$

  4. $39629.37$


Correct Option: B,D
Explanation:

Given, a=3567, b=10, c=10, d=1000, e=10000
$\frac{a}{b}+\frac{a}{c}-\frac{a}{d}+\frac{a}{e}$put the values a, b, c, d , e
in the above expression, we get
$=356.7+35.67-3.567+0.3597$
$=389.1597$

For $a = 4$, it is known that the value of the fraction $\dfrac{(a+2)x + a^2-1}{ax-2a +18}$ is independent of $x$. The other values of a for which this is the case, belong to the interval 

  1. $[-\infty, -2]$

  2. $[-2, 0]$

  3. $[0, 2]$

  4. $[2, 4]$

  5. $[4, +\infty]$


Correct Option: A
Explanation:

$\cfrac { \left( a+2 \right) x+{ a }^{ 2 }-1 }{ ax-2a+18 } $ is independent of x.

$\cfrac { a+2 }{ a } =\cfrac { { a }^{ 2 }-1 }{ 18-2a } \ 18a+36-2{ a }^{ 2 }-4a={ a }^{ 3 }-a\ { a }^{ 3 }+2{ a }^{ 2 }-15a-36=0\ { a }^{ 3 }-4{ a }^{ 2 }+6{ a }^{ 2 }-24a+9a-36=0\ (a-4)({ a }^{ 2 }+6a+9)=0\ \therefore a=-3,-3$ 
Other values of a belongs to $(-\infty ,-2]$

If $a=1\frac{3}{4}$ and $b=1\frac{2}{3}$ then the false statement is

  1. $\frac{a}{b}=\frac{b}{a}$

  2. $a\div b \neq b \div a$

  3. $a\times b=b\times a$

  4. $a\div b=ab$


Correct Option: A,D
Explanation:

$\frac{a}{b}=\frac{\frac{7}{4}}{\frac{5}{3}}$
$=\frac{21}{20}$

$\frac{b}{a}=\frac{\frac{5}{3}}{\frac{7}{4}}$
$=\frac{20}{21}$

Now check all the options

If $\dfrac{p}{q}=\bigg( \dfrac{2}{3}\bigg)^3 \div \bigg( \dfrac{3}{2}\bigg)^{-3}$ then the value of $\bigg( \dfrac{p}{q}\bigg)^{-10}=.............$

  1. $1$

  2. $0$

  3. $-1$

  4. none of these


Correct Option: A
Explanation:
  $\dfrac{p}{q}=\left ( \dfrac{2}{3} \right )^{3}\times \left ( \dfrac{3}{2} \right )^{3}$

  $\Rightarrow \dfrac{p}{q}=1$ 

  $\Rightarrow (7)^{-10}$

  $\Rightarrow 1$
- Hide questions